Respiratory System Anatomy and Physiology

  1. The breastbone spans from which vertebral levels?




    B) T2-T10

    The breastbone is synonymous with sternum.
  2. Which of the following is not true about the angle of Louis?




    C) It articulates with the first rib.

    The angle of Louis, otherwise known as the sternal angle, articulates with the second rib.
  3. Which of the following is not true about the thoracic inlet?




    C) It is larger than the thoracic outlet.

    The thoracic inlet is smaller than the thoracic outlet.
  4. Which of the following is not a part of the upper respiratory tract




    D) Trachea

    The trachea, bronchi, bronchioles, and alveoli comprise the lower respiratory tract.
  5. Which of the following is not a part of the conducting zone?




    C) None of these

    • The conducting zone consists of:
    •    1. nose
    •    2. pharynx
    •    3. larynx
    •    4. trachea
    •    5. primary, secondary, and tertiary bronchi
    •    6. terminal bronchioles

    • The respiratory zone consists of:
    •    1. respiratory bronchioles
    •    2. alveolar ducts
    •    3. alveolar sacs
    •    4. alveoli
  6. All of the following are lined with cilia, except:




    D) Alveolar ducts

    The alveolar ducts, alveolar sacs, and alveoli are not lined with cilia.
  7. All of the following contain smooth muscle, except:




    B) Alveoli
  8. All of the following are considered cartilaginous airways except:




    A) Pharynx

    The trachea, lobar bronchi, segmental bronchi, and subsegmental bronchi are classified as cartilaginous airways, whereas, the bronchioles and the terminal bronchioles are classified as noncartilaginous airways.
  9. Every 3 to 5 seconds, nerve impulses stimulate ventilation, which moves air through a series of passages into and out of the lungs. After this, there is an exchange of gases between the lungs and the blood, a process known as internal respiration.




    D) Only the first statement is true.

    After this, there is an exchange of gases between the lungs and the blood, a process known as external respiration.
  10. Air, like other gases, flows from a region with higher pressure to a region with lower pressure. Muscular breathing movements and recoil of elastic tissues create the changes in pressure that result in respiration.




    B) Only the first statement is true.

    Muscular breathing movements and recoil of elastic tissues create the changes in pressure that result in ventilation.
  11. Pulmonary ventilation involves the following pressures, except:




    C) Intraabdominal pressure

    Intraalveolar pressure is also known as intrapulmonary presusre.
  12. Active phase/s of ventilation




    C) Inspiration

    Expiration mainly occurs via relaxation of the diaphragm and passive recoil of the elastic tissues.
  13. Lungs usually reach their maximumin capacity in ___________ and decline with age after that.




    B) Early adulthood
  14. The nose consists of bone and cartilage. Two small nasal bones and extensions of the zygomatic bones form the bridge of the nose.




    D) Only the first statement is true.

    Two small nasal bones and extensions of the maxillae form the bridge of the nose.
  15. Paranasal sinuses are air-filled cavities found within the following cranial bones, except:




    C) Parietal

    Paranasal sinuses are air-filled cavities in the frontal, maxillary, ethmoid, and sphenoid bones.
  16. The paranasal sinuses:




    B) All of these
  17. Also known as hypopharynx




    C) Laryngopharynx
  18. The pharynx is a passageway that extends from the base of the skull to the level of which vertebrae?




    D) C6
  19. The laryngopharynx extends from the:




    C) Hyoid bone to the lower margin of the larynx
  20. The Waldeyer's ring consists of the following structures, except:




    C) Laryngeal tonsils

    Waldeyer's ring is a ring of lymphoid tissue that consists of adenoid (nasopharyngeal), tubal, palatine, and lingual tonsils.

    Image Upload 2
  21. Which of the following is not true about the larynx?




    D) It is formed by ten cartilages that are connected to each other by muscles and ligaments.

    The larynx is formed by nine cartilages that are connected to each other by muscles and ligaments.
  22. The false vocal cords have no role in speech production. However, they help close off the larynx when food is swallowed.




    C) Both statements are true.
  23. Which of the following structures is considered as the main airway to the lungs?




    B) Trachea

    Trachea is also known as windpipe.
  24. Adam's apple




    C) Thyroid cartilage
  25. This structure acts like a trapdoor to keep food and other particles from entering the larynx.




    A) Epiglottis
  26. The trachea divides into the left and right primary bronchi at the level of which vertebra?




    A) T5
  27. The fibrous cartilage in the tracheal wall provides support and prevents the trachea from collapsing. The posterior soft tissue allows for expansion of the esophagus, which is immediately posterior to the trachea.




    C) Only the second statement is true.

    The hyaline cartilage in the tracheal wall provides support and prevents the trachea from collapsing.
  28. The mucus membrane that lines the trachea is classified as ciliated pseudostratified columnar epithelium. This type of mucosa is also seen in the oral cavity and oropharynx.




    A) Only the first statement is true.

    This type of mucosa is also seen in the nasal cavity and nasopharynx.
  29. The mucosa of the trachea contains Goblet cells that secrete mucus. The cilia propels the mucus upward, where it is either swallowed or expelled.




    B) Both statements are true.
  30. In the mediastinum, at the level of the seventh thoracic vertebra, the trachea divides into the right and left primary bronchi. The cartilage and mucus membrane of the primary bronchi are similar to that of the trachea.




    D) Only the second statement is true.

    In the mediastinum, at the level of the fifth thoracic vertebra, the trachea divides into the right and left primary bronchi.
  31. The bronchi branch into smaller and smaller passageways until they terminate in tiny air sacs called alveoli. As the branching continues through the bronchial tree, the amount of hyaline cartilage in the walls increases, whereas the amount of the smooth muscle decreases.




    C) Only the first statement is true.

    As the branching continues through the bronchial tree, the amount of hyaline cartilage in the walls decreases, whereas the amount of the smooth muscle increases.
  32. The alveolar ducts and alveoli consist primarily of simple squamous epithelium, which permits rapid diffusion of oxygen and carbon monoxide. Exchange of gases between the air in the lungs and the blood in the capillaries occurs across the walls of the alveolar ducts and alveoli.




    D) Both statements are false.

    The alveolar ducts and alveoli consist primarily of simple squamous epithelium, which permits rapid diffusion of oxygen and carbon dioxide.
  33. The lungs contain all the components of the bronchial tree beyond the primary bronchi. The lungs are soft and spongy because they are mostly air spaces surrounded by the alveolar cells and fibrous connective tissue.




    C) Only the first statement is true.

    The lungs are soft and spongy because they are mostly air spaces surrounded by the alveolar cells and elastic connective tissue.
  34. The two lungs are separated from each other by the:




    B) Mediastinum

    The two lungs are separated from each other by the mediastinum, which contains the heart.
  35. The only point of attachment for each lung is at the:




    B) Hilum

    The only point of attachment for each lung is at the hilum, or root, on the medial side. This is also where the bronchi, blood vessels, lymphatics, and nerves enter the lungs.
  36. Which of the following is not a characteristic of the right lung?




    B) Each lobe is supplied by one of the tertiary bronchi.

    Each lobe is supplied by one of the secondary bronchi.
  37. The left lung:




    A) Has a cardiac notch on its medial side for the apex of the heart

    The left lung is longer and narrower than the right lung. It has only two lobes.
  38. At the _____________, the visceral pleura of the lungs are continuous with the parietal pleura.




    C) Hilum

    The visceral pleura is double-layered serous membrane that is firmly attached to the surface of the lungs. Meanwhile, the parietal pleura lines the wall of the thorax.
  39. The pleural cavity contains a thin film of serous fluid that is produced by the pleura. This fluid:




    A) Both of these
  40. 0th generation




    D) Trachea

    Airway generation Structure
    0 Trachea
    1 Main bronchi
    2 Lobar bronchi
    3-4 Segmental bronchi
    5-11 Subsegmental bronchi
    12-15 Bronchioles
    16 Terminal bronchioles
    17-19 Respiratory bronchioles
    20-22 Alveolar ducts
    23 Alveolar sacs
  41. Marks the transition between the conducting and respiratory zones




    D) 17th generation
  42. All of the following are responsible for the cough reflex, except:




    D) Trachea

    The first to seventh airway generations are responsible for the cough reflex.
  43. The nose contains conchae and capillaries, which warm and humidify the air. The nose bridge also assists in humidifying the air.




    D) Both statements are true.
  44. The nasopharynx is the most superior aspect of the throat and it lies posterior to the nasal cavity. The nasopharynx connects to the inner ear through the Eustachian tubes, which equalize the pressure between the ear and the nasal cavity.




    D) Only the first statement is true.

    The nasopharynx connects to the middle ear through the Eustachian tubes, which equalize the pressure between the ear and the nasal cavity.
  45. Part of the pharynx which serves as a passageway for both food and air




    A) Oropharynx
  46. The oropharynx houses the following structures, except:




    A) Adenoid tonsils

    The adenoid tonsils are found in the nasopharynx.
  47. Which of the following is not a paired cartilage of the larynx?




    B) Cricoid

    The thyroid, cricoid, and epiglottis are unpaired cartilages.
  48. Which cartilage of the larynx serves as the landmark during emergency airway puncture?




    D) Cricoid
  49. The only cartilage which completely encircles the trachea is the:




    D) Cricoid
  50. Which cartilage of the larynx is particularly prone to obstruction?




    A) Cricoid

    The cricoid cartilage is prone to obstruction due to its circular shape.
  51. The two halves of this cartilage meet in the middle to form a V-shaped anterior projection called the laryngeal prominence.




    B) Thyroid cartilage
  52. The pitch of voice depends on the angle formed by this cartilage in the larynx.




    C) Thyroid

    The thyroid cartilage acquires its shield-like shape from the embryologic midline fusion of the two distinct quadrilateral laminae.

    The laminae join at an angle of 120° in females and at 90° in males. The smaller thyroid angle explains the greater laryngeal prominence (“Adam's apple”), longer vocal cords, and lower-pitched voice in males.
  53. The mucosa above the vocal cords is classified as keratinized, stratified, squamous epithelium. Meanwhile, the mucosa below the vocal cords is classified as pseudostratified, columnar epithelium with Goblet cells and cilia.




    C) Only the second statement is true.

    The mucosa above the vocal cords is classified as nonkeratinized, stratified, squamous epithelium. This is to allow for smooth passage of food.
  54. The cilia lining the respiratory tract propel the mucus towards which structure?




    B) Pharynx

    The mucus propelled towards the pharynx is either swallowed or expelled.
  55. Which of the following is not true about the trachea?




    A) It consists of 18 to 20 C-shaped rings, which are made up of hyaline cartilage.

    The trachea consists of 16 to 20 C-shaped rings, which are made up of hyaline cartilage.
  56. In tracheostomy, a small incision is done at the _______________ down to the outer wall of the trachea.




    D) Inferior border of cricoid cartilage
  57. Which of the following cartilages acts as a protective shield surrounding the anterior part of the larynx and spans vertically from the superior to the inferior regions.




    C) Thyroid
  58. What is the largest cartilage of the larynx?




    D) Thyroid
  59. The cricoid cartilage sits in the inferior part of the larynx, at the level of the sixth cervical vertebra. This cartilage has two parts: the arch which is found anteriorly, and the lamina which is found posteriorly.




    D) Both statements are true.

    The lamina is much wider than the arch.
  60. The epiglottis is an elastic, cartilaginous, leaf-shaped flap covering the opening of the larynx. It is attached to the external surface of the thyroid cartilage and projects over the pharynx, allowing the passage of air into the larynx, trachea, and lungs.




    A) Only the first statement is true.

    The epiglottis is attached to the internal surface of the thyroid cartilage and projects over the pharynx, allowing the passage of air into the larynx, trachea, and lungs.
  61. As this structure rises during swallowing, it draws the larynx upwards to allow the passage of food or drink into the esophagus.




    B) Hyoid
  62. Pair of small, hard but flexible pyramid-shaped cartilages that sit over the posterior portion of the cricoid cartilage




    C) Arytenoid

    Image Upload 4
  63. Cartilages of Santorini




    D) Corniculate

    The corniculate cartilages are small, elastic, cone-shaped cartilages that articulate with the apices of the arytenoid cartilages.
  64. These cartilages do not directly attach to any other laryngeal cartilage but are suspended within and strengthen a fibro-elastic membrane called the aryepiglottic membrane.




    D) Cuneiform

    The cuneiform cartilages, also known as the Wrisberg cartilages, are two elongated fibrous pieces of yellow cartilage placed one on either side of the aryepiglottic fold.

    Image Upload 6
  65. Which of the following is not true about the joints of the larynx?




    A) The cricothyroid joint connects the inferior horn of the thyroid cartilage to the lamina of the cricoid cartilage.

    The cricothyroid joint connects the inferior horn of the thyroid cartilage to the arch of the cricoid cartilage.
  66. The movements in these joints change the length and tension of the vocal cords which results in modulation of the human voice.




    B) Cricothyroid joints

    Rotational movements of the arytenoid cartilages at the cricoarytenoid joints can separate (abduct) the vocal folds, widening the rima glottidis or appose (adduct) the vocal folds, narrowing the rima glottidis.
  67. Organ of phonation




    B) Larynx
  68. The thyroid cartilage is the largest of the laryngeal cartilages and is composed of hyaline cartilage. It forms the anterior and lateral portions of the larynx, but has no posterior component.




    B) Both statements are true.
  69. Which of the following is incorrect about the cricoid cartilage?




    B) Its narrow anterior arch has a midline ridge that serves as a surface of attachment for the esophagus.

    The cricoid cartilage has a narrow anterior arch (band portion) and a wider posterior lamina (signet portion) with a midline ridge that serves as a surface of attachment for the esophagus.
  70. During swallowing, as the larynx moves upward and backward, the epiglottis swings downward to close off the laryngeal inlet. This action prevents materials from entering the airway.




    B) Only the second statement is true.

    During swallowing, as the larynx moves upward and forward, the epiglottis swings downward to close off the laryngeal inlet.
  71. The epiglottis, is a large leaf-shaped elastic cartilage that is covered by mucous membrane. It is attached by its stalk to the outer aspect of the angle formed by the laminae of the thyroid cartilage via the thyroepiglottic ligament in the midline.




    C) Only the first statement is true.

    The epiglottis is attached by its stalk to the inner aspect of the angle formed by the laminae of the thyroid cartilage via the thyroepiglottic ligament in the midline.
  72. On the posterior aspect of the aryepiglottic folds, these cartilages are seen as small nodules surrounding the laryngeal inlet.




    C) Both
  73. The extrinsic laryngeal muscles move the larynx as a whole, and they consist of the suprahyoid and infrahyoid muscles. The intrinsic laryngeal muscles are responsible for moving various components of the larynx, such as modifying the length and tension of the vocal cords or the shape of the rima glottidis.




    B) Both statements are true.
  74. Newborns and infants can breathe through their nose while swallowing milk at the same time. This is possible because their larynx sits higher, and with further elevation during swallowing, the epiglottis slides up behind the hard palate, locking the larynx into the nasopharynx.




    C) Only the first statement is true.

    This is possible because their larynx sits higher, and with further elevation during swallowing, the epiglottis slides up behind the soft palate, locking the larynx into the nasopharynx.

    This anatomical arrangement which allows babies to feed and breathe at the same time is lost between 2 to 6 years of age, as the larynx gradually descends into the adult location.
  75. Which part of the pharynx has the palatoglossal arch  act as its anterior landmark?




    D) Oropharynx
  76. Which part of the pharynx transitions into the esophagus?




    B) Laryngopharynx
  77. Young children who present with a cleft palate have a failure of the two parts of the maxillary bone to unite in the midline. This causes problems with the separation of which structures, resulting in feeding and breathing issues?




    C) Nasopharynx and oropharynx
  78. The Eustachian tube serves to keep the pressure in the middle ear equalized with the pressure in the nasopharynx in relation to changes of altitude or air pressure. In case of a blockage or dysfunction of the Eustachian tube, pressure difference builds up and causes symptoms such as fullness in the ears, discomfort and hearing problems. Which of the following muscles is involved in opening of the Eustachian tube?




    A) Levator veli palatini
  79. The second pharyngeal pouch contains primitive tonsils which develop into which tonsils seen in adulthood?




    D) Palatine tonsil
  80. Which pharyngeal muscle is covered by the mucosa of the nasopharynx?




    A) Superior pharyngeal constrictor
  81. Which part of the airways extends from the uvula to the level of the hyoid bone?




    A) Oropharynx
  82. Which pharyngeal muscles are covered by the mucosa of the oropharynx?




    C) Superior and middle pharyngeal constrictors
  83. What is the name of the conical projection of the soft palate that hangs in the oropharynx?




    D) Uvula
  84. A 5-year-old girl is brought to the hospital with symptoms of ear pain, fever and irritability. Her parents report that she recently had an upper respiratory tract infection. Her doctor diagnoses an otitis media and explains to the parents that a virus or bacteria had most likely spread from her nasopharynx to her middle ear and caused the inflammation. Which of the following structures connects the middle ear with the nasopharynx?




    B) Auditory tube
  85. An 8-year-old boy is brought to a pediatrician's office suffering from difficulty of nasal breathing and phonation. After examination the doctor diagnoses him with enlarged adenoids. Which of the following anatomical structures is therefore hypertrophied?




    B) Pharyngeal tonsil
  86. Which structure marks the inferior end of the nasopharynx?




    C) Soft palate
  87. Velopharyngeal insufficiency is a condition caused due to the failure in closing communication between the oropharynx and which structure?




    A) Nasopharynx
  88. An 8-year-old boy is brought to a pediatrician's office suffering from difficulty of nasal breathing and phonation. After examination he is diagnosed with hypertrophied adenoids. Where are the affected structures located?




    A) Nasopharynx
  89. Which of the following structures of the respiratory tract does not contain the respiratory epithelium with goblet cells?




    C) Terminal bronchiole
  90. Which cells make up the alveolar sac and allow gas exchange between the alveoli and the capillaries?




    B) Type I pneumocytes
  91. A 78-year-old woman with a body mass index (BMI) of 33 kg/m2 and a history of heavy cigarette smoking presents with sudden onset tachycardia, shortness of breath and dizziness. On physical examination the physician notices swelling of her left calf. Computer tomography (CT) shows a pulmonary embolism and will therefore most likely reveal an occlusion in which of the following structures?




    D) Pulmonary artery
  92. Which of the following components comprise the alveolar septum?




    B) Endothelium and type I pneumocyte
  93. A 14-year-old girl wakes up with a dry cough and a very rapid breathing. Her mother notices that her lips are turning blue. Because she was diagnosed with allergic asthma last month, the mother immediately applies a short-acting bronchodilator prescribed by her daughter's pediatrician, which resolves the symptoms. Which cells are the main factors in the pathogenesis of this allergic reaction?




    C) Mast cells
  94. A 25-year-old man is brought to hospital after inhaling large amounts of smoke during a fire that broke out in his building. He suffers from respiratory irritation and severe cough. Which of the following cells are responsible for clearing the airways from particles that have evaded the mechanical defenses of the respiratory tract?




    B) Alveolar macrophages
  95. Which cells, along with type 2 pneumocytes, contribute to the production of surfactant in the lower respiratory tract?




    A) Exocrine bronchiolar cells
  96. Which type of epithelium is present in the terminal bronchiole?




    C) Simple ciliated columnar epithelium
  97. Shortly after birth, a neonate born prematurely after 28 weeks of gestation presents with respiratory difficulties, tachycardia and cyanosis. After a chest radiograph showed low lung volume and small infiltrates, the diagnosis of neonatal respiratory distress syndrome is established. This condition is due to deficiency in surfactant, which decreases alveolar surface tension. Which of the following cells are responsible for surfactant secretion?




    B) Type II pneumocytes
  98. A 55-year-old woman presents to the emergency department with acute onset of shortness of breath. Her symptoms last for 2 days and have progressively worsened. She has been a smoker for approximately 20 years, smoking about two packs of cigarettes per day. After performing a chest X-ray and lung function tests, the doctor establishes the diagnosis of emphysema. The destruction of the following structures occur in the pathophysiologic mechanism of emphysema, except:




    A) Bronchiole
  99. Which of the following airway structures lack cartilage in their wall?




    B) Respiratory bronchiole
  100. What type of epithelium lines the lumen of the trachea?




    D) Ciliated pseudostratified columnar epithelium with Goblet cells
  101. Which cells of the trachea protect the airways by removing small inhaled particles from the lumen?




    B) Ciliated cells
  102. Asthma is a type of obstructive inflammatory disease characterized by smooth muscle hypertrophy of the airways, and mucus plugging. Which of the following airway structures contain both goblet cells, responsible for mucus production, and smooth muscle cells?




    B) Lobar bronchi
  103. Which structures of the trachea produce and secrete mucus in the airway?




    C) Tracheal gland and Goblet cells
  104. All of the following tissues are located between the submucosa and tunica adventitia of the trachea, except:




    B) Elastic cartilage
  105. Which tissue comprises the majority of the tracheal submucosa?




    A) Dense irregular connective tissue
  106. Which layers comprise the mucosa of the trachea?




    D) Respiratory epithelium and lamina propria
  107. Which of the following cells can be found in the mucosal layer of the trachea?




    C)  Ciliated cells and Goblet cells
  108. Which cells of the respiratory tract increase in number with chronic exposure to irritants such as smoking?




    D) Goblet cells
  109. Which of the following tissues does not connect the free ends of the incomplete cartilage rings of the trachea?




    A) Dense connective tissue
  110. Which structure is in charge of keeping the lumen of the trachea open?




    D) Cartilage
  111. Which layer of the tracheal wall contains numerous mucus-secreting glands?




    D) Submucosa
  112. In which layer of the tracheal wall are the tracheal glands usually located?




    C) Submucosa
  113. Which layers the tracheal wall contain lymphatic tissue?




    C) Lamina propria and submucosa
  114. Which cells of the trachea protect the airways by removing small inhaled particles from the lumen?




    A) Ciliated cells
  115. A 6-year old boy is brought by his parents to the doctor's office. They complain that their son is constantly coughing and had multiple respiratory and ear infections since birth. After performing a chest x-ray, the physician notes an inverted positioning of the internal organs (situs invertus). After further diagnostics, the so-called Kartagener Syndrome is diagnosed. Dysfunction of which of the following structures is responsible for the symptoms above-mentioned?




    C) Cilia
  116. Which epithelial cell specialization actively moves mucus and foreign bodies out of the respiratory tract?




    B) Cilia
  117. Which structure separates the tracheal epithelium from the lamina propria?




    B) Basement membrane
  118. Which of the following is not a layer of the tracheal wall?




    D) Elastic cartilage
  119. The lungs have no attachment to the walls of the chest cage, except where it is suspended at its hilum from the mediastinum. Intermittent suction of excess pleural fluid into the lymphatic channels maintains slight suction between parietal and visceral pleurae.




    C) Only the first statement is true.

    Continual suction of excess pleural fluid into the lymphatic channels maintains slight suction between parietal and visceral pleurae.
  120. Parietal pleura is sensitive to:




    B) Pain

    Meanwhile, visceral pleura is sensitive to stretch.
  121. Air-filled alveoli will appear as black or hollowed on a chest radiograph. The presence of increased opacity or white colored flecks could be indicative of tissue scarring or secretion buildup.




    A) Both statements are true.
  122. All pulmonary volumes and capacities are usually about 20% to 25% less in men than in women. These measurements are also less in small and asthenic people than in large and athletic people.




    A) Only the second statement is true.

    All pulmonary volumes and capacities are usually about 20% to 25% less in women than in men.
  123. What partial pressure of oxygen in the arterial blood necessitates oxygen supplementation?




    B) 55 mmHg

    The normal partial pressure of oxygen in the arterial blood ranges between 80 to 100 mmHg.
  124. Oxygen supplementation is needed if the oxygen saturation falls below:




    B) 88%

    Normal oxygen saturation is 95% to 100%.
  125. What is the primary rhythm generator for respiration?




    D) Dorsal respiratory group of neurons

    The pneumotaxic and apneustic centers in the pons regulates the rate of respiration. The ventral respiratory group of neurons in the medulla controls voluntary expiration and acts to increase the force of inspiration.
  126. What is the most gravity dependent region of the lung where greatest perfusion occurs?




    B) Zone III

    In zone I, the ventilation occurs in excess of perfusion. In zone II, the perfusion and ventilation are fairly equal.
  127. A therapist listens to the lung sounds of a 56-year old male with chronic bronchitis. The patient was admitted to the hospital 2 days ago after complaining of shortness of breath and difficulty breathing. While performing auscultation, the therapist identifies distinct lung sounds with a relatively high, constant pitch during exhalation. This type of sound is most consistent with?




    D) Wheezes

    Crackles or rales is discontinuous bubbling sound secondary to the presence of secretions. Rhonchi is a snoring sound secondary to mild narrowing of the airway.
  128. Following a hard tackle, a football quarterback exhibits signs of fractured ribs and pneumothorax. Upon auscultation over the injured area, the therapist would expect?




    C) Decreased or no breath sounds

    Soft, rustling sound on inhalation are vesicular breath sounds which are normally heard in a healthy lung. Crackles are heard due to increased mucus production, whereas wheezes are heard because of airway narrowing. In pneumothorax, there is decreased or absent breath sounds because the presence of air in the pleural space increases the distance between the lungs and the thoracic wall, making the breath sounds difficult to hear.
  129. Which of the following is not a function of the surfactant?




    A) None of these
  130. A low lung compliance would mean that the lungs would need greater-than-average change in intrapleural pressure to change the volume of the lungs. Compliance is directly related to the elastic recoil of the lungs, so thickening of the lung tissue will decrease lung compliance.




    C) Only the first statement is true.

    Compliance is inversely related to the elastic recoil of the lungs, so thickening of the lung tissue will decrease lung compliance.
  131. When blood pH drops then the amount of oxyhemoglobin ____________ and oxygen delivery to the tissue cells ____________.




    B) Decreases; increases

    This scenario describes the Bohr effect. It describes hemoglobin's lower affinity to oxygen secondary to increases in partial pressure of carbon dioxide and/or decreased blood pH. This lower affinity, in turn, enhances the unloading of oxygen into the tissues to meet their oxygen demands.
  132. The following statements concerning the main bronchi are correct, except:




    C) The right main bronchus gives off the superior lobar bronchus after entering the hilum of the lung.

    The right main bronchus gives off the superior lobar bronchus before entering the hilum of the lung, whereas the left main bronchus gives off the superior lobar bronchus after entering.
  133. The following statements concerning the lungs are correct, except:




    C) The cardiac notch lies in the lower lobe of the left lung.

    The cardiac notch lies in the upper lobe of the left lung.
  134. What is the most important pathway for the respiratory response to systemic arterial CO2 (PcO2)?




    B) H+ activation of chemosensitive area of medulla

    CO2 is the major controller of respiration as a result of direct effect of H+ on the chemosensitive area of medulla. H+ do not cross the blood-brain barrier; thus, the CO2 diffuses across the said barrier and is then converted into H+, which acts on the chemosensitive area of medulla. CO2 and H+ activation of carotid bodies are minimal under normal conditions.
  135. The apneustic center transmits signals to the dorsal respiratory group of neurons to "switch off" the inspiratory signals, thus controlling the duration of the filling phase of the lung cycle. This has a secondary effect of increased rate of breathing because limitation of inspiration also limits expiration as well as the entire respiration.




    B) Only the second statement is true.

    The pneumotaxic center transmits signals to the dorsal respiratory group of neurons to "switch off" the inspiratory signals, thus controlling the duration of the filling phase of the lung cycle.
  136. When the respiratory drive for increased pulmonary ventilation becomes greater than normal, a special set of respiratory neurons that are inactive during normal quiet breathing then becomes active, contributing to the respiratory drive. These neurons are located in:




    B) Ventral medulla

    The basic rhythm of respiration is generated in the dorsal group of respiratory neurons, which is located almost entirely within the nucleus of tractus solitarius. When the respiratory drive for increased pulmonary ventilation becomes greater than normal, respiratory signals spill over into the ventral group, causing this group to also contribute to the respiratory drive. However, the ventral group of respiratory neurons remain almost totally inactive during normal quiet breathing.
  137. The afferent ending for Hering-Breuer reflex are mechanoreceptors located in the:




    C) Bronchi and bronchioles

    The Hering-Breuer reflex mechanoreceptors are located in the bronchi and bronchioles and respond to increased stretch to limit respiration.
  138. Horizontal fissure is seen on:




    A) Right lung

    Horizontal fissure is seen on right lung, whereas oblique fissure is seen on both lungs.
  139. Bronchiectasis is a progressive form of obstructive lung disease characterized by irreversible destruction and dilation of airways generally associated with chronic viral infections. The most immediate symptom of this condition is persistent coughing with large amounts of purulent sputum, which is worse in the evening.




    B) Both statements are false.

    Bronchiectasis is a progressive form of obstructive lung disease characterized by irreversible destruction and dilation of airways generally associated with chronic bacterial infections. The most immediate symptom of this condition is persistent coughing with large amounts of purulent sputum, which is worse in the morning.
  140. Pleurisy is the inflammation of the pleura caused by viral or bacterial infection, injury, or tumor. The symptoms develop gradually, usually with a sharp, sticking chest pain that is worse on inspiration, coughing, and sneezing.




    D) Only the first statement is true.

    The symptoms develop suddenly, usually with a sharp, sticking chest pain that is worse on inspiration, coughing, and sneezing. The visceral pleura is insensitive to pain, so pain associated with pleurisy most likely comes from the inflammation of parietal pleura.
  141. During full remission, patients with asthma are asymptomatic and their pulmonary function tests are normal. At the beginning of an attack, there is a sensation of chest constriction, inspiratory and expiratory wheezing, nonproductive coughing, shortened expiration, tachycardia, and tachypnea.




    A) Only the first statement is true.

    At the beginning of an attack, there is a sensation of chest constriction, inspiratory and expiratory wheezing, nonproductive coughing, prolonged expiration, tachycardia, and tachypnea.
  142. The primary area affected in cystic fibrosis is:




    B) Pancreas

    Approximately 90% of patients with cystic fibrosis have pancreatic insufficiency with thick secretions blocking the pancreatic ducts, causing dilation of the small lobes of the pancreas, degeneration, and eventual progressive fibrosis throughout.
  143. Non-continuous breath sound described as a soda pop fizzing or hair rubbed through the finger next to the ear




    D) Rales

    Wheezes are continuous breath sounds heard during exhalation and are comparable to a musical tone or a whistling sound. A pleural friction rub is a raspy breath sound caused by inflammation of the pleura.
  144. During the examination of a patient, a therapist notes hypoventilation, muscular twitching, and increased deep tendon reflexes. Which of the following conditions is the most likely cause of the patient's signs and symptoms?




    B) Metabolic alkalosis

    Metabolic and respiratory acidosis would result in decreased deep tendon reflexes. Respiratory alkalosis results in tachypnea.
  145. The proper sequence of events that occurs during coughing is:




    A) Inspiration, closing of glottis, tightening of vocal cords, abdominal contraction, opening of glottis, expiration
  146. Manual vibration requires the palmar aspect of the therapist's hands to be in full contact with the affected lung segment. Vibration is usually applied at the beginning of deep inspiration and is maintained throughout the end of expiration.




    A) Only the first statement is true.

    Vibration is usually applied at the end of deep inspiration and is maintained throughout the end of expiration.
  147. The respiratory tract is divided into two sections at the level of the:




    C) Vocal cords

    The respiratory tract is divided into two sections at the level of the vocal cords; the upper and lower respiratory tract.
  148. The lungs are most often considered as part of the lower respiratory tract, but are sometimes described as a separate entity. They contain the terminal bronchioles, alveolar ducts, alveolar sacs and alveoli.




    B) Only the first statement is true.

    The lungs contain the respiratory bronchioles, alveolar ducts, alveolar sacs and alveoli.
  149. Which of the following is incorrect about the upper respiratory tract?




    D) It lies above the thyroid cartilage and vocal cords.

    The upper respiratory tract lies above the cricoid cartilage and vocal cords.
  150. Which of the following structures inside the nasal cavity disrupt the laminar flow of air, making it slow and turbulent, thereby helping to humidify and warm up the air to body temperature.




    D) Nasal conchae

    The nasal conchae are bony projections found at the lateral walls of the nasal cavity.
  151. The oropharynx is a pathway for both the air incoming from the nasopharynx and the food incoming from the oral cavity. Thus, the oropharynx is lined with the more protective:




    D) Non-keratinized squamous epithelium
  152. It is the region of the pharynx at which the respiratory and digestive systems diverge.




    C) Laryngopharynx

    Anteriorly, the laryngopharynx continues into the larynx, whereas posteriorly it continues as the esophagus.
  153. This structure represents the trunk of the tracheobronchial tree.




    D) Traches
  154. The apex of the lung extends into the root of the neck, whereas the base of the lung rests upon the diaphragm.




    A) Both statements are true.
  155. Each hilum of the lung contains the following structures, except:




    D) Somatic plexus

    Each hilum of the lung contains the following structures: principal bronchi, pulmonary artery, pulmonary veins, pulmonary autonomic plexus, bronchial vessels, and lymph nodes and vessels.
  156. The epithelium of the terminal bronchioles contains exocrine bronchiolar cells called:




    A) Club cells

    Club cells, which are formerly known as Clara cells, are non-ciliated cuboidal cells that contribute to the production of surfactant.
  157. Type I pneumocytes are thin, squamous cells that carry out the gas exchange, while type II pneumocytes are larger cuboidal cells that produce surfactant.




    A) Both statements are true.
  158. What is the most common type of upper respiratory tract infection?




    D) Common cold

    The common cold is a viral infection that usually involves the nose and the throat.
  159. You are asked to review a 63-year-old female who was admitted with shortness of breath. On your arrival, the patient appears drowsy and is on 10L of oxygen via a mask.

    You perform an ABG, which reveals the following results:

    PaO2: 7.0 kPa or 52.5 mmHg 
    pH: 7.29
    PaCO2: 9.1 kPa or 68.2 mmHg 
    HCO3–: 26 mEq/L
    Base excess: +1

    Which of the following statements is incorrect based on the findings above?

    A) In the context of low PaO2, a raised PaCO2 suggests the patient has type 1 respiratory failure.
    B) The PaCO2 is raised significantly and this is likely to be the
    cause of the acidosis.
    C) The HCO3- is normal, so the metabolic system is not contributing to the acidosis and is also not compensating for the respiratory acidosis, suggesting that this is an acute derangement.
    D) The base excess is within normal limits as there has been no significant change in the amount of HCO3-.
    A) In the context of low PaO2, a raised PaCO2 suggests the patient has type 1 respiratory failure.

    Type I respiratory failure (hypoxemic) involves low arterial oxygen pressure (PaO2) and normal or low arterial carbon dioxide pressure (PaCO2). Meanwhile, type II respiratory failure (hypercapnic) involves low PaO2 and high PaCO2.

      Normal Values
    pH 7.35-7.45
    PaO2 11-13 kPa or 82.5-97.5 mmHg
    PCO2 4.7-6.0 kPa or 35.2-45 mmHg
    HCO3- 22-26 mEq/L
    Base excess -2 to +2
  160. All of the following symptoms are associated with hypercapnia, except:




    D) Weak pulse

    It should be bounding pulse.
  161. Which of the following is not a possible cause of type 2 respiratory failure?




    C) Acute bronchitis

    • The following are possible causes of type 2 respiratory failure
    •    1. Increased airways resistance (e.g. emphysema, asthma)
    •    2. Reduced breathing effort (e.g. opiate use, brain stem lesion, extreme obesity)
    •    3. Decrease in the area of the lung available for gas exchange (e.g. chronic bronchitis)
    •    4. Neuromuscular problems (e.g. Guillain-Barré syndrome, motor neuron disease)
    •    5. Deformity (e.g. ankylosing spondylitis, flail chest)
  162. A 17-year-old patient presents to A&E complaining of a tight feeling in their chest, shortness of breath as well as some tingling in their fingers and around their mouth. They have no significant past medical history and are not on any regular medication. An ABG is performed on the patient whilst they’re breathing room air and the results are shown below:

    PaO2: 14 kPa or 105 mmHg
    pH: 7.49
    PaCO2: 3.2 kPa or  24 mmHg 
    HCO3–: 22 mEq/L
    BE: +2

    What does the ABG show?




    B) Respiratory alkalosis

      Normal Values
    pH 7.35-7.45
    PaO2 11-13 kPa or 82.5-97.5 mmHg
    PCO2 4.7-6.0 kPa or 35.2-45 mmHg
    HCO3- 22-26 mEq/L
    Base excess -2 to +2
  163. A 17-year-old patient presents to A&E complaining of a tight feeling in their chest, shortness of breath as well as some tingling in their fingers and around their mouth. They have no significant past medical history and are not on any regular medication. An ABG is performed on the patient whilst they’re breathing room air and the results are shown below:

    PaO2: 14 kPa or 105 mmHg
    pH: 7.49
    PaCO2: 3.2 kPa or  24 mmHg 
    HCO3–: 22 mEq/L
    BE: +2

    Which of the following statements below is incorrect?




    C) The bicarbonate is on the low end of normal, representing compensation to the acid-base imbalance.

    The bicarbonate is on the low end of normal, but this does not represent compensation. Compensation would involve a much more significant reduction in HCO3–.

      Normal Values
    pH 7.35-7.45
    PaO2 11-13 kPa or 82.5-97.5 mmHg
    PCO2 4.7-6.0 kPa or 35.2-45 mmHg
    HCO3- 22-26 mEq/L
    Base excess -2 to +2
  164. All of the following can cause respiratory alkalosis, except:




    C) Insufficient mechanical ventilation

    • The following are possible causes of respiratory alkalosis:
    •    1. Anxiety (e.g. panic attack)
    •    2. Pain (causes increased respiratory rate)
    •    3. Hypoxia (often seen in ascent to altitude)
    •    4. Pulmonary embolism
    •    5. Pneumothorax
    •    6. Iatrogenic (e.g. excessive mechanical ventilation)
  165. Which of the following is not true about perioral and peripheral paresthesia felt during hyperventilation?




    D) As blood plasma becomes alkalotic, the concentration of biologically active component of blood calcium increases.

    As blood plasma becomes more alkalotic, the concentration of freely ionized calcium, the biologically active component of blood calcium, decreases. This hypocalcemia is responsible for the paraesthesia often seen with hyperventilation.
  166. A 48-year-old male has been admitted with a 24 hour history of abdominal distention and profuse vomiting. A CT scan reveals a large mass causing bowel obstruction. As part of the patient’s assessment, the surgical registrar requests that you check his blood gas (on air), with the results shown below:

    PaO2: 12.7 kPa or 95.2 mmHg 
    pH: 7.50 
    PaCO2: 5.5 kPa or 41 mmHg
    HCO3-: 29 mEq/L
    BE: +3

    Which of the following statements below is not true given the above findings?




    D) The respiratory system can attempt to compensate for a metabolic alkalosis by increasing ventilation.

    The respiratory system can attempt to compensate for a metabolic alkalosis by increasing PaCO2 or by decreasing ventilation. However, in the short term, the respiratory system will likely maintain PaCO2 within the normal range.

      Normal Values
    pH 7.35-7.45
    PaO2 11-13 kPa or 82.5-97.5 mmHg
    PCO2 4.7-6.0 kPa or 35.2-45 mmHg
    HCO3- 22-26 mEq/L
    Base excess -2 to +2
  167. A 48-year-old male has been admitted with a 24 hour history of abdominal distention and profuse vomiting. A CT scan reveals a large mass causing bowel obstruction. As part of the patient’s assessment, the surgical registrar requests that you check his blood gas (on air), with the results shown below:

    PaO2: 12.7 kPa or 95.2 mmHg 
    pH: 7.50 
    PaCO2: 5.5 kPa or 41 mmHg
    HCO3-: 29 mEq/L
    BE: +3

    What is the most likely cause of such ABG findings?




    B) History of profuse vomiting


    As a result of this patient’s profuse vomiting, he has lost significant amounts of HCL, a stomach acid. This results in a net loss of H+ ions, meaning less H+ to bind to HCO3– and therefore more free HCO3– in the system.

    In addition, as a result of vomiting, the patient is volume depleted, which results in the release of aldosterone and other mineralocorticoids which in turn increase HCO3– reabsorption by the kidneys, further increasing the amount of free HCO3– in the serum.

      Normal Values
    pH 7.35-7.45
    PaO2 11-13 kPa or 82.5-97.5 mmHg
    PCO2 4.7-6.0 kPa or 35.2-45 mmHg
    HCO3- 22-26 mEq/L
    Base excess -2 to +2
  168. You are asked to review a 59-year-old female who has been admitted the acute medical ward of your hospital. The nurse tells you that she appears short of breath despite currently receiving 3 liters of oxygen via nasal cannula.

    You take an arterial blood gas which reveals the following results:

    PaO2: 9.1 kPa or 68.2 mmHg 
    pH: 7.30
    PaCO2: 8.4 kPa or 63 mmHg 
    HCO3-: 29 mEq/L
    BE: +4

    All of the following statements are correct, except:




    A) An oxygen flow rate of 3L via nasal cannulae would be expected to deliver an inspired concentration (FiO2) of around 32%, therefore you would expect that the PaO2 would be approximately 20 kPa less than this.

    • An oxygen flow rate of 3L via nasal cannulae would be expected to deliver an inspired concentration (FiO2) of around 32%, therefore you would expect that the PaO2 would be approximately 10 kPa less than this (e.g. 22 kPa). Therefore, 
    • a PaO2 of 9.1 kPa is therefore grossly abnormal and indicates significant hypoxia.

      Normal Values
    pH 7.35-7.45
    PaO2 11-13 kPa or 82.5-97.5 mmHg
    PCO2 4.7-6.0 kPa or 35.2-45 mmHg
    HCO3- 22-26 mEq/L
    Base excess -2 to +2
  169. You are asked to review a 59-year-old female who has been admitted the acute medical ward of your hospital. The nurse tells you that she appears short of breath despite currently receiving 3 liters of oxygen via nasal cannula.

    You take an arterial blood gas which reveals the following results:

    PaO2: 9.1 kPa or 68.2 mmHg 
    pH: 7.30
    PaCO2: 8.4 kPa or 63 mmHg 
    HCO3-: 29 mEq/L
    BE: +4

    Which of the following statements below is incorrect?




    A) If this derangement in CO2 was acute, there would have been time for a compensatory response from the metabolic system.

    If this derangement in CO2 was acute, there would not have been time for a compensatory response from the metabolic system.

      Normal Values
    pH 7.35-7.45
    PaO2 11-13 kPa or 82.5-97.5 mmHg
    PCO2 4.7-6.0 kPa or 35.2-45 mmHg
    HCO3- 22-26 mEq/L
    Base excess -2 to +2
  170. An 89-year-old patient presents with fever, rigors, hypotension and reduced urine output. They appear confused and are unable to provide any meaningful history. The care home that the patient came from has provided some basic documentation. You look through the information available and note that the district nurse changed this patient’s catheter 24 hours ago. The medical registrar commences antibiotics, aggressive fluid resuscitation and asks you to perform an arterial blood gas, with the results shown below. The patient was not on oxygen at the time of the ABG.

    PaO2: 12.4 kPa or 93 mmHg
    pH: 7.29 
    PaCO2: 5.5 kPa or 41.2 mmHg 
    HCO3-: 15 mEq/L
    BE: – 4

    Which of the following statements below is not consistent with the ABG findings?




    B) There is an evidence of respiratory compensation for the metabolic acidosis.

    The PaCO2 is normal, so there is no evidence of respiratory compensation for the metabolic acidosis. A lower than normal PaCO2 suggest respiratory compression.

      Normal Values
    pH 7.35-7.45
    PaO2 11-13 kPa or 82.5-97.5 mmHg
    PCO2 4.7-6.0 kPa or 35.2-45 mmHg
    HCO3- 22-26 mEq/L
    Base excess -2 to +2
  171. An 89-year-old patient presents with fever, rigors, hypotension and reduced urine output. They appear confused and are unable to provide any meaningful history. The care home that the patient came from has provided some basic documentation. You look through the information available and note that the district nurse changed this patient’s catheter 24 hours ago. The medical registrar commences antibiotics, aggressive fluid resuscitation and asks you to perform an arterial blood gas, with the results shown below. The patient was not on oxygen at the time of the ABG.

    PaO2: 12.4 kPa or 93 mmHg
    pH: 7.29 
    PaCO2: 5.5 kPa or 41.2 mmHg 
    HCO3-: 15 mEq/L
    BE: – 4

    What is the most likely diagnosis given the patient's history and ABG analysis?




    A) Urosepsis

    Urosepsis is the most likely diagnosis given the clinical presentation and history of recent catheter change.
  172. An 89-year-old patient presents with fever, rigors, hypotension and reduced urine output. They appear confused and are unable to provide any meaningful history. The care home that the patient came from has provided some basic documentation. You look through the information available and note that the district nurse changed this patient’s catheter 24 hours ago. The medical registrar commences antibiotics, aggressive fluid resuscitation and asks you to perform an arterial blood gas, with the results shown below. The patient was not on oxygen at the time of the ABG.

    PaO2: 12.4 kPa or 93 mmHg
    pH: 7.29 
    PaCO2: 5.5 kPa or 41.2 mmHg 
    HCO3-: 15 mEq/L
    BE: – 4

    Which of the following is not true about this patient's condition?




    A) Anaerobic respiration produces uric acid as a byproduct, which has resulted in the addition of acid to the patient’s serum causing uremic acidosis.

    Anaerobic respiration produces lactic acid as a byproduct, which has resulted in the addition of acid to the patient’s serum causing lactic acidosis.
  173. A 22-year-old female is brought into A&E by ambulance with a 5-day history of vomiting and lethargy. When you begin to talk with the patient you note that she appears disorientated and looks clinically dehydrated. At present, you are unable to gain any further details, but the patient looks very unwell from the end of the bed. You gain IV access, send off a routine panel of bloods and commence some fluids.  You ask the nurse to check the patient’s observations and she notes an increased respiratory rate, low blood pressure and tachycardia. You perform an ABG on the advice of your registrar. The results of the ABG are shown below (the patient was not on oxygen when this was taken).

    PaO2: 13 kPa or 97.5 mmHg
    pH: 7.3 
    PaCO2: 4.1 kPa or 30.7 mmHg
    HCO3-: 13 mEq/L
    BE: -4

    Which of the following statements below is incorrect given the ABG findings?




    C) There is no evidence of respiratory compensation.

    There is an evidence of respiratory compensation as the value of PaCO2 is lower than normal.

      Normal Values
    pH 7.35-7.45
    PaO2 11-13 kPa or 82.5-97.5 mmHg
    PCO2 4.7-6.0 kPa or 35.2-45 mmHg
    HCO3- 22-26 mEq/L
    Base excess -2 to +2
  174. Which of the following is not true about diabetic ketoacidosis?




    A) The body attempts to compensate for the metabolic acidosis by hyperventilating to blow off CO2 and thereby decrease pH.

    • The body attempts to compensate for the metabolic acidosis by hyperventilating to blow off CO2 and thereby increase pH
    • This hyperventilation, in its extreme form, may be observed as Kussmaul respiration.
  175. A 56-year-old man was found unconscious at home with a respiratory rate of 6 breaths per minute and pinprick pupils. An ambulance was called and the paramedics administered some naloxone. On arrival to A&E his ABG showed the following (not on oxygen at the time of the ABG):

    PaO2: 7.9 kPa or 59 mmHg
    pH: 7.31 
    PaCO2: 7.1 or 53 mmHg 
    HCO3-: 22 mEq/L
    BE: +1 

    Which of the following statements below is not true given the above findings?




    A) Base excess is lower than the normal range.

    Base excess is within the normal range, indicating the absence of metabolic compensation.

      Normal Values
    pH 7.35-7.45
    PaO2 11-13 kPa or 82.5-97.5 mmHg
    PCO2 4.7-6.0 kPa or 35.2-45 mmHg
    HCO3- 22-26 mEq/L
    Base excess -2 to +2
  176. A 56-year-old man was found unconscious at home with a respiratory rate of 6 breaths per minute and pinprick pupils. An ambulance was called and the paramedics administered some naloxone. On arrival to A&E his ABG showed the following (not on oxygen at the time of the ABG):

    PaO2: 7.9 kPa or 59 mmHg
    pH: 7.31 
    PaCO2: 7.1 or 53 mmHg 
    HCO3-: 22 mEq/L
    BE: +1 

    What is the most likely cause of this patient's acid-base derangement?




    B) Opiate 

    The history suggests this man has taken an overdose of opioids given the reduced level of consciousness, respiratory depression and pinpoint pupils. The respiratory depression has led to hypoxia, hypercapnia, and ultimately respiratory acidosis.

      Normal Values
    pH 7.35-7.45
    PaO2 11-13 kPa or 82.5-97.5 mmHg
    PCO2 4.7-6.0 kPa or 35.2-45 mmHg
    HCO3- 22-26 mEq/L
    Base excess -2 to +2
  177. A 77-year-old lady was admitted to hospital 10 days ago with a fractured neck of femur. The orthopedic team repaired the fracture and she has been an inpatient on the orthopedic ward recovering ever since. The patient’s nurse is becoming increasingly concerned as the patient’s oxygen requirements are increasing (she is now on 3L) and the patient is now tachypneic (respiratory rate 35). In addition, the patient has recently started complaining of calf pain.

    You review the patient and perform an ABG which reveals the following:

    PaO2: 6 kPa or 45 mmHg
    pH: 7.51 
    PaCO2: 3.1 kPa or 23.2 mmHg 
    HCO3-: 21 mEq/L
    BE: 0 

    Which of the following statements below is not consistent with the above findings?




    A) The patient has respiratory alkalosis and type II respiratory failure.

    The patient has respiratory alkalosis and type I respiratory failure, as evidenced by her low PaO2 and PaCO2.

      Normal Values
    pH 7.35-7.45
    PaO2 11-13 kPa or 82.5-97.5 mmHg
    PCO2 4.7-6.0 kPa or 35.2-45 mmHg
    HCO3- 22-26 mEq/L
    Base excess -2 to +2
  178. A 77-year-old lady was admitted to hospital 10 days ago with a fractured neck of femur. The orthopedic team repaired the fracture and she has been an inpatient on the orthopedic ward recovering ever since. The patient’s nurse is becoming increasingly concerned as the patient’s oxygen requirements are increasing (she is now on 3L) and the patient is now tachypneic (respiratory rate 35). In addition, the patient has recently started complaining of calf pain.

    You review the patient and perform an ABG which reveals the following:

    PaO2: 6 kPa or 45 mmHg
    pH: 7.51 
    PaCO2: 3.1 kPa or 23.2 mmHg 
    HCO3-: 21 mEq/L
    BE: 0 

    What is the likely diagnosis, given the patient's history and the above results?




    B) Pulmonary embolus

    From the clinical history, this patient may have a deep vein thrombosis and a secondary pulmonary embolus. This has resulted in an increased oxygen requirement.

    The patient is likely hyperventilating to maintain her oxygenation level, but as a result is exhaling excessive amounts of CO2, leading to alkalosis.

      Normal Values
    pH 7.35-7.45
    PaO2 11-13 kPa or 82.5-97.5 mmHg
    PCO2 4.7-6.0 kPa or 35.2-45 mmHg
    HCO3- 22-26 mEq/L
    Base excess -2 to +2
  179. A 24-year-old medical student has just returned from his elective in Ghana. In the last few days, he has developed severe diarrhea and has now presented to A&E. On assessment, he is very dehydrated and tachypneic.

    An ABG is performed and reveals the following:
    PaO2: 14.6 kPa or 109.5 mmHg 
    pH: 7.32 
    PaCO2: 4.0 kPa or 30 mmHg 
    HCO3-: 13 mEq/L
    BE: -4 

    What is the most accurate interpretation of the above findings?




    C) Metabolic acidosis with respiratory compensation

    A pH of 7.32 is low, suggesting acidosis. The HCO3- and the base excess are low, so the metabolic system most likely contributes to the acidosis. The PaCO2 is low, therefore the respiratory system is not contributing to the acidosis; however, a low PaCO2 indicates that the respiratory system is compensating for the acidosis.

      Normal Values
    pH 7.35-7.45
    PaO2 11-13 kPa or 82.5-97.5 mmHg
    PCO2 4.7-6.0 kPa or 35.2-45 mmHg
    HCO3- 22-26 mEq/L
    Base excess -2 to +2
  180. A 64-year-old man is admitted to A&E with central crushing chest pain. As the nurses are getting him attached to the ECG he has a cardiac arrest. Thankfully, CPR was commenced immediately and after 6 minutes he regained spontaneous circulation and began breathing again.

    An ABG (on 15L O2) performed following this sequence of events reveals the following:

    PaO2: 9.5 kPa or 71.3 mmHg
    pH: 7.14 
    PaCO2: 8.1 kPa or 60.8 mmHg 
    HCO3-: 15.2 mEq/L
    BE: – 9.7 

    Which of the following statements below is incorrect?




    B) This patient had a cardiac arrest which meant there was no period of impaired ventilation and end-organ perfusion.

    • This patient had a cardiac arrest which meant there was a period of impaired ventilation and end-organ perfusion.
    •  
    • The hypercapnia caused respiratory acidosis, whereas the accumulation of products of anaerobic respiration (as a result of hypoxia and reduced end-organ perfusion) caused metabolic acidosis.
  181. True or False: When swallowing, the soft palate would have to move downward in order to prevent food from entering the lungs.
    False

    When swallowing, the soft palate would have to move upward in order to prevent food from entering the nose.
  182. True or False: The smooth muscle within the vocal cords allows them to change their tension for vocalization.
    False

    The elastic tissues within the vocal cords allow them to change their tension for vocalization.
  183. True or False: The trachea contains rings of cartilage that surround the windpipe to keep it rigid.
    False

    The trachea contains rings of cartilage that surround the windpipe to keep it semirigid.
  184. True or False: The lungs contain a ciliated mucous lining that helps remove foreign particles from them.
    False

    The tracheobronchial tree contains a ciliated mucous lining that helps remove foreign particles from them.
  185. True or False: The respiratory membrane is comprised of the two layers of the alveoli.
    False

    The respiratory membrane is comprised of an alveolar wall, a capillary wall, and their basement membranes.
  186. The exchange of gases between the blood and the cells of the body is called:




    B) Internal respiration

    Internal respiration is also known as tissue respiration.
  187. Which of these functions cannot be attributed to the mucous membrane that lines the nasal cavity?




    A) Serves as the site for initial gas exchange between blood and air

    Gas exchange only occurs in the alveoli.
  188. The cartilage marks the lowermost portion of the larynx.




    D) Cricoid

    The cricoid cartilage is approximately at the level of C6.
  189. The true vocal cords are those that produce the sounds of the human voice. Where are these located?




    C) Inferior to the false vocal cords

    The false vocal folds are located at the inferior edge of the vestibule just above the true vocal folds. The space or gap between the two vocal cords is known as the rima glottidis.
  190. Within the bronchial tree, which tubes are small branches that enter the lobules of the lungs?




    C) Intralobular bronchioles
  191. Thin-walled outpouchings of the alveolar ducts of the lungs are called:




    B) Alveolar sacs
  192. The primary force responsible for air moving into the lungs during inhalation is:




    B) Atmospheric pressure
  193. The lipoprotein known as _____________ reduces surface tension within the alveoli so they do not collapse during exhalation.




    D) Surfactant
  194. The force of exhalation is primarily due to:




    C) Elastic recoil of thoracic muscles
  195. Even after the most forceful exhalation, a certain volume of air remains in the lungs. This volume is called the:




    B) Residual volume
  196. The maximum amount of air a person can exhale after taking the deepest breath possible is the:




    C) Vital capacity
  197. The basic rhythm of breathing is controlled by which portion of the brain?




    C) Dorsal respiratory group of the medullary rhythmicity center
  198. The respiratory membrane, across which gases diffuse, is very thin and is made up of:




    D) Alveolar cell membrane, capillary membrane, and fused basement membranes
  199. Whether oxygen diffuses from the alveoli into the blood or not is dependent upon:




    B) The partial pressure of oxygen in the lungs and in the blood
  200. Within the blood, oxygen is primarily transported __________________ while the bulk of carbon dioxide is transported _________________.




    A) As oxyhemoglobin; as bicarbonate ions
  201. What is the first structure in this respiratory sequence?




    C) Pharynx

    Pharynx is also known as the throat.
  202. Which structures play the greatest role in warming and humidifying air?




    B) Conchae

    The nasal conchae or turbinates are long, narrow curled shelves of bone that protrude into the nasal cavity. The superior, middle and inferior conchae divide the nasal cavity into four groove-like air passages.
  203. The area directly superior to the soft palate is the:




    C) Nasopharynx
  204. Which sinuses are not paranasal sinuses?




    D) Mastoid

    It should be maxillary.
  205. Where are the pharyngeal tonsils located?




    C) Nasopharynx

    The nasopharynx contains the pharyngeal (adenoid) tonsils, whereas the oropharynx contains the lingual and palatine (faucial) tonsils.
  206. Which passageway serves as a common route for food or air?




    D) Oropharynx
  207. The opening to the larynx called?




    D) Glottis
  208. Which part of the respiratory tree has the greatest relative percentage of smooth muscle fibers?




    A) Bronchioles
  209. The only structures that allow gas diffusion across them are the:




    A) Alveoli
  210. Which principle normally prevents the alveoli from collapsing during expiration?




    D) Secretion of surfactant
  211. The space between the two lungs is called the:




    B) Mediastinum
  212. What are the most powerful stimuli for breathing?




    C) Low pH; high CO2
  213. True or False: In order to maintain homeostasis, chloride ions follow as bicarbonate moves out of the erythrocyte.
    False

    In the chloride shift, Cl- ions are exchanged for HCO3- in order to maintain a state of electrical balance (electrophysiologic homeostasis).
  214. True or False: The main purpose for cellular oxygen is to provide a means of producing most of the cell energy.
    True

    Oxygen is used in many metabolic steps to result in energy, including the citric acid cycle. Only a small amount of energy can be obtained anaerobically by human cells.
  215. True or False: During the act of swallowing the epiglottis moves upward and the glottis appears as a triangular slit.
    False

    In swallowing, the epiglottis moves downward and the laryngeal muscles close the glottis to prevent food from entering the trachea.
  216. True or False: The alveoli exchange gases between the air and the capillaries within them.
    False

    The capillaries are found outside of the alveoli and do not come into direct contact with the air.
  217. True or False: The diaphragm is a dome-shaped smooth muscle organ innervated by the phrenic nerves.
    False

    The diaphragm is comprised of skeletal muscle which is under voluntary and involuntary control mechanisms.
  218. True or False: The surface tension associated with the lungs can either cause the lungs to expand or collapse.
    True

    The surface tension between the pleura allow the lungs to fully expand while the surface tension within the alveoli could cause their collapse.
  219. True or False: Respiratory distress syndrome in an infant results from an inability to breathe because of excess surfactant production in the alveoli by type I cells.
    False

    Respiratory distress syndrome, which is otherwise known as hyaline membrane disease, results from an inability to produce a surfactant from type II alveolar cells.
  220. True or False: During inspiration the diaphragm moves upward while thorax expands and moves outward.
    False

    During inspiration, the diaphragm contracts and moves downward, causing the thorax to increase and the abdomen to decrease in size.
  221. True or False: When the diaphragm contracts the pressure within the thoracic cavity decreases.
    True

    When the diaphragm contracts the pressure within the thoracic cavity falls because of the larger space, and is now below the atmospheric.
  222. True or False: The compliance of the lung increases as the lungs expand.
    False

    The compliance of the lung decreases as the lungs expand.
  223. The anatomical dead space represents the air that is trapped in the alveoli and is around 150cc.
  224. True or False: The anatomical dead space represents the air that is trapped in the alveoli and is around 150cc.
    False

    The anatomical dead space represents the amount of air that does not come in contact with the alveoli but remains in the passages.
  225. True or False: The alveolar dead space is a volume of air that cannot cross the alveoli because of a disease process.
    True

    The alveolar dead space is the amount of air that cannot functionally cross the alveoli for exchange because of changes such as poor capillary blood flow.
  226. True or False: Bronchogenic carcinoma is a lung cancer that has spread or metastasized from some outside organ.
    False

    Bronchogenic carcinoma begins in the epithelial lining of the bronchial tree.
  227. True or False: The alveolar ventilation rate is usually less than the minute respiratory volume.
    True

    The alveolar ventilation rate is the breathing rate per minute times the tidal volume minus the physiologic dead space. In a male, this is around 4,200cc per minute in comparison to 6,000 cc per minute which is an average minute respiratory volume.
  228. True or False: There are chemoreceptors in the aortic and carotid bodies which are very sensitive to fluctuations in the levels of blood oxygen.
    False

    Oxygen is a vital gas required for cells but the body is least able to detect it directly, but instead relies on reflexes associated with pH or carbon dioxide. The direct response to low oxygen levels is weak.
  229. True or False: Decreasing blood oxygen and increasing carbon dioxide cause the major increases seen in breathing rate during strenuous exercise.
    False

    Studies have now indicated that the stretch reflexes mediated by proprioceptors in the joints and muscles may have a greater effect on increasing heart rate than blood parameters.
  230. True or False: Hyperventilation causes its symptoms mainly because it results in respiratory acidosis.
    False

    Hyperventilation causes the excess elimination of carbon dioxide and H+ ions, therefore raising the pH. Alkalosis then causes CNS changes such as dizziness.
  231. True or False: The respiratory membrane is the simple squamous epithelium of the alveoli.
    False

    The respiratory membrane is the combination of the alveolar membrane and capillary membrane in contact with it.
  232. True or False: There are phagocytic cells within the lungs that move among the alveoli to remove foreign particles.
    True

    The alveolar macrophages are able to move through the pores in the alveoli and are part of the reticuloendothelial system.
  233. True or False: If the alveolar partial pressure of carbon dioxide was 45 mm Hg, carbon dioxide would not readily diffuse into the air.
    True

    The movement of gases depends upon simple laws of diffusion; the capillary PCO2 is usually 45 mm Hg so there would be no net movement in this case.
  234. True or False: The PO2 in the pulmonary veins is less than the PO2 in the alveolar air.
    True

    The alveolar PO2 is 104 mm Hg while the pulmonary blood that leaves the lung, in the pulmonary veins, has a PO2 of 100 mm Hg.
  235. True or False: Retrolental fibroplasia develops in an infant who does not have sufficient blood oxygen.
    False

    Retrolental fibroplasia may develop and cause blindness in infants who have been exposed to excess oxygen or hyperoxia conditions, because of retinal capillary damage from the oxygen.
  236. True or False: ARDS is a condition of lung collapse or atelectasis in a newborn.
    False

    Atelectasis can occur in the infant respiratory distress syndrome. ARDS is the adult respiratory distress syndrome.
  237. True or False: At high altitudes it is difficult to breathe because the air has a lower percent of oxygen.
    False

    At high altitudes, the percent of oxygen may remain the same but there is a lower effective pressure because the atmospheric pressure and therefore the partial pressure is less than 160 mm Hg.
  238. True or False: Oxygen dissociates faster from hemoglobin during exercise.
    True

    Hemoglobin releases its oxygen faster under conditions of acidity such as those generated during skeletal muscle contraction. The purpose is to increase muscle oxygen.
  239. True or False: Carbon dioxide and oxygen compete for binding sites with the hemoglobin.
    False

    Carbon dioxide is transported on the NH2 of the globin as carbaminohemoglobin and oxygen is independently carried on the iron.
  240. The utilization of oxygen by metabolic processes is correctly referred to as:




    B) Cell respiration

    During cell (tissue) respiration, oxygen enters the cell and carbon dioxide is eliminated. During this same exchange, intercellular metabolic reactions are also using oxygen to produce energy.
  241. Which of the following structures is a part of the upper respiratory tract?




    A) Pharynx

    The lower respiratory tract includes the respiratory organs within the thorax and the upper respiratory tract includes those outside the thorax such as the nasal cavity.
  242. The internal nares will open into the:




    B) Nasal pharynx

    The external nares are the nostrils that open into the vestibule of the nasal cavity and the internal nares are the exit of the nasal cavity into the pharynx.
  243. The olfactory nerves pass through openings in the:




    D) Cribriform plate

    The olfactory nerves for smell pass through the foramina of the cribriform plate of the ethmoid bone on the superior nasal surface.
  244. Most of the respiratory passage surface is lined with:




    C) Pseudostratified epithelium

    The entrance into the system is covered with stratified squamous epithelium which is followed by pseudostratified columnar epithelium.
  245. Which of the following is not a property of the mucous lining of the respiratory tract?




    C) Contains microvilli

    Microvilli are cell extensions found in the gastrointestinal tract. Cilia move in a wavelike manner to remove foreign particles in the respiratory tract.
  246. Which of the following does not contain a sinus?




    A) Zygomatic

    The zygomatic bone is the solid cheek bone, whereas the frontal, ethmoid, sphenoid, and maxillary bones have mucus filled sinus cavities.
  247. What is the anatomical term for the throat?




    B) Pharynx

    The pharynx is the throat which opens into the larynx or voicebox.
  248. Where are the palatine tonsils located?




    A) Oral pharynx

    The palatine tonsils are within the fauces of the soft palate of the oral pharynx. The adenoids (pharyngeal tonsils) are in the nasal pharynx.
  249. The epiglottis is part of the __________ and covers the glottis.




    C) Larynx

    The epiglottis is comprised of a mucous membrane-lined piece of cartilage that acts as a cover over the glottis of the larynx to prevent food from entering the trachea.
  250. The true vocal cords move because they are attached to the ___________ cartilage.




    D) Arytenoid

    The largest piece of cartilage is the thyroid.
  251. C-shaped pieces of cartilage are found within the:




    D) Trachea

    The trachea, which is also known as windpipe, contains C-shaped pieces of cartilage which allow the soft trachea to remain open.
  252. The carina separates the _____________ from each other.




    C) Primary bronchi

    The carina is an area of cartilage which separates the openings of the two primary bronchi.
  253. There are _____ secondary bronchi.




    D) 5

    There are two secondary bronchi on the left and three on the right.
  254. The bronchioles enter the _________ of the lungs.




    C) Lobules

    The bronchioles enter the basic subdivision areas called the lobules. The respiratory bronchioles enter the alveoli.
  255. The functional gas exchanging units are termed:




    B) Alveoli

    The terminal sacs called the alveoli are the only areas specialized for capillary gas exchanges.
  256. The relative amount of smooth muscle is highest in the:




    C) Bronchioles

    The bronchioles have lost their cartilage and become mainly comprised of smooth muscle which causes them to collapse during the spasms of asthma.
  257. The tissue lining the alveoli is ________________ epithelium.




    C) Simple squamous

    The respiratory bronchioles are lined with cuboidal cells which changes to simple squamous epithelium in the alveoli for diffusion.
  258. How many alveoli are there in an average adult lung?




    C) 300,000,000

    The average lung has around 300 million alveoli that represent a surface of about 70-80 square meters.
  259. Gas is exchanged in the alveoli by a process of:




    A) Simple diffusion

    The main force that governs the movement of gases is the rate of diffusion which results from concentration differences.
  260. What is the entrance into the lungs called?




    A) Hilus

    The hilus is the region that represents the door into the lung, whereas the apex is the narrowed superior surface.
  261. The pleural cavity is the space between the lung and the:




    B) Parietal pleura

    The pleural cavity is the serous fluid-filled potential space between the two layers of the pleural membranes, namely the visceral and parietal pleurae.
  262. The main function of the pleural membranes is to:




    B) Reduce friction

    The pleural membranes are serous and slippery and allow the lungs to slide during breathing without damage from friction rubs.
  263. Atmospheric pressure is around _____ mmHg.




    A) 760

    Atmospheric pressure at sea level is 760 mmHg or 1 atmosphere or 760 torr.
  264. The movement of air within the respiratory system is correctly referred to as:




    A) Ventilation

    Ventilation is the movement of air throughout a duct system, while respiration is the exchange of gases. Breathing includes the mechanical events involved.
  265. What prevents the alveoli from collapsing during exhalation?




    B) Phospholipids

    Surfactant is a phospholipid. Surface tension, acting alone, would cause collapse.
  266. The main force causing the exhalation of air is the:




    D) Elastic recoil

    Normal exhalation is said to be passive because muscles are not contracting. The elastic tissue in the lungs should be sufficient to compress the lungs.
  267. During inspiration the intra-alveolar pressure reaches ______ mmHg




    D) 758

    During inspiration, the thorax expands and the intra-pulmonary pressure falls below atmospheric. This negative pressure causes air to enter the lungs.
  268. A pneumothorax occurs when the thoracic pleural pressure reaches around _____ mmHg.




    A) 760

    A pneumothorax occurs whenever the pleural pressure reaches atmospheric, causing the lung to collapse. This may result from injury or disease.
  269. The ______________ muscle can cause a forced exhalation.




    C) Internal intercostal

    The internal intercostal muscles between the ribs cause the cage to move downward and inward during forced exhalation.
  270. The amount of air that enters the lungs during normal, restful breathing is called the:




    C) Tidal volume

    The tidal volume is around 500cc in an adult male and somewhat less in a female.
  271. The amount of air that cannot be expelled is the:




    C) Residual volume

    The residual volume is around 1,000cc and cannot be removed without collapsing the lungs.
  272. Which of these is the sum of the other three?




    D) Vital capacity

    The vital capacity represents the deepest possible breath a person could take.
  273. The total amount of air that cannot be exchanged either because of disease or because it cannot reach an exchange site is termed the:




    C) Physiologic dead space

    The physiologic dead space is the sum of the anatomic and alveolar dead space volumes.
  274. The condition characterized by alveolar fibrosis with difficulty exhaling is:




    D) Emphysema

    Emphysema involves loss of the elastic recoil capabilities of the lungs due to the replacement of normal alveolar tissue with fibrous scar tissue. Smoke is a major precipitating factor.
  275. The condition characterized by difficulty breathing due to an allergic response is:




    A) Bronchial asthma

    Some persons develop an allergic reaction to foreign particles that results in severe episodes of bronchial smooth muscle spasms causing a restriction in airway flow.
  276. Non-respiratory movements are most easily triggered by stimuli in which of these areas?




    D) Larynx

    The distal parts of the lower respiratory tract lack the type of nerve supply that can act as a stimulus pathway to elicit a cough.
  277. Which reflex involves a spasmodic contraction of the diaphragm?




    C) Hiccup

    A hiccup is the sudden spastic contraction of the diaphragm and results in a sound of air striking the vocal cords. Reflexes such as these are associated with neurological and blood gas alterations.
  278. Which reflex is caused by local alveolar decreases in oxygen content?




    D) Yawn

    The yawn is a deep inhalation which is a reflex attempt to correct high carbon dioxide or low oxygen levels in the blood. The cause of hiccups is unknown.
  279. Breathing is mainly controlled by the:




    B) Medulla oblongata

    The medulla oblongata in the brain stem contains the rhythmicity areas that coordinate the incoming stimuli with outgoing responses (breathing).
  280. Which area exerts its effect on the primary breathing control areas?




    B) Pneumotaxic area of pons

    The most primary or vital centers for breathing are located within the medulla. The pons acts as an accessory center to modify the rate of medullary breathing impulses.
  281. Which of the following factors favors an increase in breathing rate?




    B) Increased carbon dioxide in the blood

    The factors that stimulate breathing are high carbon dioxide, low pH, high acidity, and low oxygen levels in the blood.
  282. Which of the following does not stimulate the chemosensitive neurons of the medulla?




    A) Low oxygen in the blood

    The brain does not normally respond to levels of oxygen directly but must rely on distal receptors in the aortic and carotid bodies. It is assumed that spinal fluid and blood pH levels should be closely similar.
  283. Which of these is not a chronic obstructive pulmonary disease?




    C) Lung cancer

    COPD is characterized by some impairment in the ability of the respiratory tree to deliver sufficient air to the sites of exchange. Lung cancer may or might not be obstructive.
  284. Which mechanism prevents over-inflation of the lungs?




    A) Hering-Breuer reflex

    There are stretch receptors in the pleural membranes and lungs that respond to excess inflation by causing reflex exhalation via the brain stain centers. One is the inflation or Hering-Breuer reflex.
  285. Which is the most common cause of hyperventilation?




    B) Anxiety

    Although an abnormal increase in breathing rate may be caused by a brain tumor or drugs, the most common cause is psychological in origin (anxiety).
  286. Which is the most abundant gas in the atmosphere?




    C) Nitrogen

    The air is 78% nitrogen, 21% oxygen, and 0.04% carbon dioxide. There is less than 0.05% argon and other gases present.
  287. What is the partial pressure of atmospheric oxygen?




    B) 160 mmHg

    The partial pressure is the percent times total pressure or 21% times 760 mm Hg or 160 mm Hg. 0.3 mm Hg is the partial pressure of CO2.
  288. What is the normal PCO2 of the capillary blood?




    A) 45 mmHg

    The partial pressure of capillary carbon dioxide is 45 mm Hg which is greater than the 40 mm Hg in the alveolar air, allowing CO2 to leave the blood.
  289. What is the normal PO2 of the alveolar air?




    C) 104 mmHg

    The PO2 of the alveolar air is 104mm Hg, while the capillary blood is 40 mm Hg, allowing oxygen to diffuse into the blood.
  290. Where is the highest amount of carbon dioxide found?




    C) Arteries entering the lung

    The pulmonary arteries carry the carbon dioxide rich blood from the tissues of the body to the site of exchange in the alveoli. Recall that CO2 is an end product of metabolism.
  291. Which condition is characterized by inflammation and infection involving the alveoli?




    B) Pneumonia

    Pneumonia is an inflammation of the lung alveoli that usually is caused by a bacterium such as Streptococcus. Tuberculosis is an infection that does not necessarily involve the alveoli.
  292. Most of the oxygen in the blood is found in the:




    D) Hemoglobin

    Oxygen can only dissolve in water to a maximum of 3%. Most oxygen, 98%, is bound in the oxyhemoglobin of the erythrocytes.
  293. At high PO2 levels, where is most oxygen located?




    A) Oxyhemoglobin

    The ability of the heme of the hemoglobin to transport oxygen is based on physical laws of pressure and attraction. The plasma holds only 1.5 to 3% of the total oxygen.
  294. Which of the following factors favors a release of oxygen from hemoglobin?




    C) Decreased pH

    Hemoglobin will release its oxygen faster under conditions of increased acidity, higher temperatures, and carbon dioxide. It is most saturated when the blood is more alkaline.
  295. Which of these has the highest affinity for hemoglobin?




    C) CO

    Carbon monoxide competes with oxygen causing carboxyhemoglobin to form. Nitrogen does not bind to hemoglobin.
  296. Where is most carbon dioxide transported?




    B) As bicarbonate

    Since CO2 readily reacts with water, bicarbonate ions are formed and represent the most common method of transporting carbon dioxide.
  297. The most important factor that allows the blood to carry carbon dioxide in high amounts is:




    C) Carbonic anhydrase

    Carbonic anhydrase is an enzyme found in erythrocytes that increases the chemical conversion of carbon dioxide into bicarbonate.
  298. What is the structure and function of type I cells in the alveoli of the lungs?





    E) Squamous cells, make up 95% cell volume and allow gas exchange

    Type I cells are thin and flat and form the structure of the alveoli. Type I alveolar cells are squamous (giving more surface area to each cell) and cover approximately 90–95% of the alveolar surface. Type I cells are involved in the process of gas exchange between the alveoli and blood. These cells are extremely thin (sometimes only 25 nm) – the electron microscope was needed to prove that all alveoli are covered with an epithelial lining. These cells need to be so thin to be readily permeable for enabling an easy gas exchange between the alveoli and the blood.
  299. Which of these would increase the diffusion capacity of the lung?




    D) Being a male

    Male sex is associated with a higher lung diffusing capacity, even after correcting for age and body size. On average, DLCO in males is about 10% greater than females.

    A smaller body size results in decreased lung diffusing capacity. This is because smaller people have smaller lungs, with a lower alveolar surface area.

    Older people have lower lung diffusing capacities. The diffusing capacity decreases by 2% per year after the age of 20.

    The diffusing capacity is higher in a supine position. This is due to the increased volume of blood flowing through alveolar capillaries in the supine position, when compared to standing.
  300. Which of the following stimulates respiration by activating central chemoreceptors?





    B) Increased arterial carbon dioxide tension

    Increased arterial carbon dioxide tension stimulates respiration by activating central chemoreceptors. In fact, it is the only stimulus to which central chemoreceptors stimulate respiration in response, whereas peripheral chemoreceptors stimulate respiration in response to reduced arterial oxygen tension, increased arterial carbon dioxide tension and increased arterial hydrogen ion concentration.

    However, in conditions where the arterial carbon dioxide tension is chronically increased, such as chronic obstructive pulmonary disease, the central chemoreceptors no longer stimulate respiration in response to increased carbon dioxide tension because they adapt to the new set point of increased arterial carbon dioxide tension. In this case, the person's respiratory drive solely depends on the peripheral chemoreceptors stimulating respiration in response to reduced arterial oxygen tension.

    This explains why people with chronic obstructive pulmonary disease have a reduced percentage oxygen saturation compared to those without it and why this reduced saturation is maintained if they are given oxygen therapy. If this oxygen saturation was increased by oxygen therapy, it would remove the sole remaining stimulus to breathe (the reduced arterial oxygen tension to which the peripheral chemoreceptors respond) and lead to apnoea
  301. Which of the following cell type in the lung is responsible for regeneration of alveolar epithelium after injury?




    B) Type 2 pneumocyte

    Type 2 pneumocytes are small cuboidal cells found in alveoli that are able to reproduce and regenerate alveolar epithelium after injury. Type 2 pneumocytes also produce surfactant that lines the alveolus.

    Type 1 pneumocytes are cells of the alveoli that facilitate gas diffusion. They are not capable to reproducing, but type 2 pneumocytes can differentiate into type 1 pneumocytes after injury.

    Pseudostratified ciliated columnar epithelium and goblet cells together comprise the respiratory epithelium which lines the airways. These are not involved in the regeneration of alveolar epithelium post-injury
  302. What is the name of the secretory cell, lining the alveolar wall, that enables gaseous exchange to effectively occur?




    A) Type 2 pneumocyte

    Type 2 pneumocystis secrete surfactant, which lowers surface tension.  Surface tension is, simply put, the propensity for H+ cations at the surface of a liquid to be drawn together - thus surface tension can be thought of as a collapsing force of airways. From week 25 (roughly), sufficient surfactant is secreted to decrease surface tension commensurate with viable respiration - babies born prior to this point require ventilatory support and corticosteroids.

    Type 1 pneumocytes are extremely thin simple squamous epithelia lining the vast majority of the alveoli. They function to provide the media through which gaseous exchange occurs between airway and red blood cell.

    Type 3 pneumocytes is an often used term to describe fixed macrophages that reside within the terminal airways, including alveoli, due to the constant risk of inhaling various pathogens.

    Type 4 pneumocytes do not exist.
  303. The majority of carbon dioxide in the blood is transported which of the following?





    E) Bicarbonate ions

    Carbon dioxide, produced by metabolizing tissues, diffuses out of cells and into blood plasma, where it is dissolved. Some of this carbon dioxide remains in the plasma however only a small proportion of the overall amount, therefore this is not the correct answer.

    Most diffuses into red blood cells, where it is converted into bicarbonate ions by the enzyme carbonic anhydrase in the following reaction: CO2 + H2O <- -=""> HCO3- + H+. These bicarbonate ions are then transported to the lungs and reconverted back into carbon dioxide that is ready to be exhaled. Therefore this is the correct answer.

    A small amount of carbon dioxide combines with hemoglobin to form carbaminohemoglobin and carboxyhemoglobin is a compound of carbon monoxide and hemoglobin. These are therefore not the correct answer.
  304. The activation of which of these receptors in the airways can lead to bronchoconstriction?




    A) J receptor

    J receptors are sensory receptors that respond to chemical and mechanical stimuli. When the lung parenchyma is irritated, stimulation of J-receptors can reflexively induce tachypnea, mucus hypersecretion and bronchoconstriction.

    Parasympathetic stimulation of bronchiolar smooth smooth muscle causes bronchoconstriction, while adrenergic stimulation results in bronchodilation due to relaxation of smooth muscle.

    The main type of parasympathetic receptor in the lung is the muscarinic receptor (the nicotinic receptor is found at the neuromuscular junction of skeletal muscle).

    Stimulation of the adrenergic B2 receptor causes bronchodilation and relaxation of the smooth muscles.

    Slow adapting pulmonary stretch receptors sense changes in lung volume, and they do not directly affect changes in airway calibre.
  305. Which of the following statements regarding oxygen diffusion in the lungs is correct?




    C) The partial pressure of oxygen in alveoli is low compared to that in the external environment.

    The partial pressure of oxygen in alveoli is low compared to that in the external environment. This is the case because carbon dioxide is simultaneously diluting the alveolar atmosphere and oxygen diffusion into capillaries is continuously occurring

    The partial pressure of oxygen in alveoli is high compared to that in capillaries which allows an overall diffusion of oxygen into the blood down a concentration gradient. Oxygen then combines with haemoglobin, after crossing alveolar and capillary membranes, to form oxyhaemoglobin. It is in this form that enables oxygen to be transported around the body.
  306. What is the structure and function of type II cells in the alveoli of the lungs?





    C) Cuboidal cells which secrete pulmonary surfactant

    Type II cells secrete pulmonary surfactant to lower the surface tension of water and allows the membrane to separate, therefore increasing its capability to exchange gases. The surfactant is continuously released by exocytosis.

    Type II alveolar cells cover a small fraction of the alveolar surface area. Type II cells are also capable of cellular division, giving rise to more type I and II alveolar cells when the lung tissue is damaged. These cells are granular and roughly cuboidal.

    Type II alveolar cells are typically found at the blood-air barrier. They make up <5% of the alveolar surface.
  307. From which vessel does the bronchial arterial circulation arise?





    A) Descending aorta

    The lung has a dual blood supply, receiving deoxygenated blood from the right ventricle via the pulmonary artery and oxygenated blood via the bronchial circulation.

    The pulmonary artery divides to accompany the bronchi.

    The arterioles accompanying the respiratory bronchioles are thin-walled and contain little smooth muscle.

    The bronchial circulation arises from the descending aorta.

    These bronchial arteries supply tissues down to the level of the respiratory bronchiole.

    The bronchial veins drain into the pulmonary vein, forming part of the normal physiological shunt.
  308. At which level in the respiratory system is the flow of air the greatest?





    D) Trachea

    From the trachea to the periphery, the airways become smaller in size (although greater in number).

    The cross-sectional area available for airflow increases as the total number of airways increases.

    The flow of air is greatest in the trachea and slows progressively towards the periphery (as the velocity of airflow depends on the ratio of flow to the cross-sectional area).

    In the terminal airways, gas flow occurs solely by diffusion.
  309. With respect to the respiratory system, which of the following values is equal to the approximate volume of air that makes up the anatomical dead space of the conducting zone?




    D) 150 ml

    The anatomical dead space refers to the air that resides in the conducting zone (from the nose or mouth down to the level of the terminal bronchioles) that does not participate in gas exchange. There is approximately 150ml in the anatomical dead space at any one time.

    The significance of this dead space is that not all the air in each breath is available for the exchange of oxygen and carbon dioxide.

    500ml is the approximate volume of air inspired at one time. Inspiration pushes the 150ml of "dead space" air from the conducting zone into the alveoli. In this way, 150ml of "old air" enter the alveoli along with 350ml of fresh inspired air and 150ml of the newly inspired air remains in the conducting zone.
  310. How does hyperventilation lead to perioral and peripheral paresthesia?




    D) Hypocalcemia

    As blood plasma becomes more alkalotic, the concentration of freely ionised calcium, the biologically active component of blood calcium, decreases (hypocalcemia).

    Because a portion of both hydrogen ions and calcium are bound to serum albumin, when blood becomes alkalotic, the bound hydrogen ions dissociate from albumin, freeing up the albumin to bind with more calcium and thereby decreasing the freely ionized portion of total serum calcium leading to hypocalcemia.

    This hypocalcemia related to alkalosis is responsible for the paresthesia often seen with hyperventilation.
  311. What is the definition of total lung capacity?





    C) The volume in the lungs at maximal inflation.

    The average total lung capacity of an adult human male is about 6 litres of air.

    Tidal breathing is normal refers to normal breathing at rest. Tidal volume is the volume of air that is inhaled or exhaled in a single breath.

    The average human respiratory rate is 30-60 breaths per minute at birth, decreasing to 12-20 breaths per minute in adults.
  312. The central chemoreceptors located in the medulla oblongata directly respond to which of the following?




    C) CSF [H+]

    The central chemoreceptors of the central nervous system, located on the ventrolateral medullary surface, respond directly to CSF [H+]. They provide 80% of the respiratory drive.

    CSF[H+] is derived via PaCO2 and is, therefore, an indirect response to PaCO2.

    Circulating [H+] cannot cross the blood-brain barrier.
  313. A patient with wet-sounding productive cough, positive "sweat test" for high chloride, failure to thrive and malnutrition is diagnosed with cystic fibrosis (CF).  

    In CF, mutations to the CTFR gene result in thickened mucous production from the mucosal cells of the respiratory tract. 

    Which of the following cell types are not present in the mucosal layer of the human airway?




    D) Simple cuboidal epithelium

    Pseudostratified ciliated columnar epithelial cells predominate the mucous layer of the respiratory tract, and are termed "respiratory epithelia".

    Stratified squamous epithelium lines the vocal ligaments (folds of mucosa lining the cricovocal ligament superiorly) due to constant abrasion, many thousands of times daily.

    Goblet cells, secreting mucus, are present throughout the respiratory tract.
  314. A 23-year-old man visiting Africa dies in a motor vehicle accident. An autopsy is performed.

    On autopsy of the lung, there is a small cluster of caseating granulomas in the right lung just above the interlobar fissure and similar granulomas are found in the hilar lymph nodes. Acid-fast staining shows acid-fast bacilli in these lesions. No other lesions are found in the remaining organs and tissues.

    Which is the most accurate interpretation of these findings?






    F) Primary Mycobacterium tuberculosis infection

    The Ghon complex is the most frequent pathologic form of primary pulmonary tuberculosis. Mycobacterium tuberculosis first localizes in the lung parenchyma, then in the hilar lymph nodes. In both of these locations, a granulomatous reaction takes place. These lesions usually heal by fibrosis, leaving only small scars at the sites of remote tuberculous infection.

    In some cases, owing to immunosuppression (e.g. AIDS, immunosuppressant treatment, and lymphomas), reactivation of dormant bacilli in old lesions or additional re-exposure leads to secondary tuberculosis, with the progression of lesions.

    Sometimes, active lesions of the Ghon complex are discovered by chance at autopsy.

    Disseminated infection is a sign of secondary tuberculosis due to lymphohematogenous dissemination and subsequent seeding of tubercle bacilli throughout the body, with myriad small granulomas forming in the lungs, spleen, liver, bone marrow, retina and adrenals.

    Reactivation disease occurs when immunosuppression or immunocompromise results in the activation of bacilli in old lesions with progression to more severe disease. Cavitary disease and miliary tuberculosis may result.

    Re-exposure disease may result in secondary tuberculosis and progression of lesions.

    Scars due to remote healed tuberculosis are frequently found postmortem and listed in autopsy reports as incidental findings.
  315. What is the function of the posterior cricoarytenoid muscle of the larynx?




    D) Vocal cord abduction

    The posterior cricoarytenoid muscles are the sole abductors of the vocal cords in the larynx.
  316. Which region of the respiratory system is normally considered sterile?





    B) Sinuses

    The pneumatic sinuses are the parts of the respiratory system that are normally sterile and free of pathogens.

    A sinus is simply a cavity within the osseous skull that contains air. There a number of sinuses within the skull, including the maxillary, frontal and sphenoidal sinuses, as well as the mastoid and ethmoidal air cells. It is thought they reduce the density (and thus, mass) of the skull without compromising structural integrity. Another speculated reason for the development of sinuses is their secretions, described below, that distinctly separate the bone adjacent to the central nervous system from the outside, non-sterile environment of the naso-, oro- and laryngopharynx, and nasal cavity. Having a physical and immunological barrier has significant benefits if viewed in this light.

    The sinuses contain mucous-secreting glandular epithelial cells. This mucous typically drains into the nasal cavity, and the retrograde movement of bacteria through these drainage foramen can lead to infection.

    There are also B lymphocytes present here, which promote the secretion of IgA as a constituent of the mucous. It is this IgA that helps to prevent both bacterial infection and proliferation of bacteria if they do enter the sinuses. Infection in the sinuses is referred to as sinusitis, and this can be life-threatening if the surrounding bone becomes infected.

    The other options - the laryngopharynx, alveoli, Eustachian tube and oral cavity - all contain bacteria or are more susceptible to pathogen invasion if there is disruption of the normal flora of that region. An infection of each of these can be a combined laryngitis and pharyngitis, pneumonia, acute otitis media due to bacterial invasion and migration into the Eustachian tube, and oral candidiasis.
  317. Which structures connect the thyroid cartilage to the arytenoid cartilages in the trachea?




    D) Vocal cords

    The arytenoid cartilages are attached to the superior aspect of the cricoid cartilage and to the inferior-posterior-medial aspect of the thyroid by the vocal cords. The arytenoid cartilages function as 'control levers' to either tense or relax the vocal cords to produce sound.
  318. Which structure attaches the thyroid gland to the thyroid and cricoid cartilages?





    E) Ligament of Berry

    The ligament of Berry is the structure connecting the thyroid gland to the thyroid and cricoid cartilages. Its other name is the suspensory ligament of the thyroid gland.

    While conventionally described as being attached to both the thyroid and cricoid cartilages, it is usually only attached to the cricoid cartilage. In a minority of cases, it can extend up to the thyroid cartilage. The ligament of Berry is responsible for the superior and inferior displacement of the thyroid gland during swallowing.

    The ligament of Treitz helps suspend the small intestine at the duodenojejunal flexure. It connects this portion of the small intestine to the connective tissue surrounding the superior mesenteric artery and the coeliac trunk.

    The investing layer of the deep cervical fascia of the neck is a fascial layer that helps to separate the neck into different layers. It encloses the entire neck, and contains the trapezius and sternocleidomastoid muscles.

    The triangle of Calot is a triangular space in the abdominal cavity formed by the inferior free edge of the liver, the common hepatic duct and the cystic duct. This space contains the cystic artery.

    The internal spermatic fascia is found within the spermatic cord.
  319. Which of the following laryngeal muscles receives motor innervation via the superior laryngeal nerve?




    A) Cricothyroid muscle

    The cricothyroid muscle receives motor innervation via the superior laryngeal nerve.

    The recurrent laryngeal nerve supplies motor innervation to the vast majority of the internal muscles of the larynx (except the cricothyroid muscle).
  320. What is the definition of the rima glottidis of the larynx?




    C) Opening between the true vocal cords and the arytenoid cartilages

    The rima glottidis can be defined as the opening between the true vocal cords and the arytenoid cartilages.

    This space is closed by the lateral cricoarytenoid muscles and the arytenoid muscles and is opened by the posterior cricoarytenoid muscles.
Author
carminaorlino
ID
356095
Card Set
Respiratory System Anatomy and Physiology
Description
Updated